You are on page 1of 57

Faraday’s Law

2
Expectations
After this Lecture, students will:

1. Understand and apply Faraday’s Law of electromagnetic induction.


2. Understand the concept of magnetic flux, and calculate the value of
a magnetic flux.
3. Understand and apply Lenz’s Law
:Question
On the previous chapters we were calculating the
magnetic field due to current carrying
.conductors
:But Now
Can a magnetic field produce an electric field
?that can induce an electric current

4
Michael Faraday
1791 – 1867
•British physicist & chemist
•Great experimental scientist

Contributions to Electricity:
• He was the first to produce
an electric current from a magnetic
field, and invented the first electric
motor and dynamo.
Faraday’s Experiment – Set Up
•A primary coil is connected
to a battery.

•No battery is present in the


secondary coil.
https://hotcore.info/act/kareff-090456.html.

•The secondary coil is not directly


connected to the primary coil.
Faraday’s Findings
•At the instant the switch is closed,
the ammeter changes from zero in one direction,
then returns to zero.
•When the switch is opened,
the ammeter changes in the opposite direction,
then returns to zero.
•The ammeter reads zero when there is a steady
current or when there is no current in the primary
circuit.
Faraday’s Experiment: Conclusions
•An electric current can be induced in a loop by a changing
magnetic field.
•The induced current exists only while the magnetic field through
the loop is changing.

•This is generally expressed as:


An induced emf is produced in the loop by the changing magnetic field.
Faraday’s Law of Induction: Lenz’s Law
•Faraday found that the induced emf in a wire loop is
Proportional to the time Rate of Change of the
Magnetic Flux Through the Loop.
•Magnetic Flux is defined similarly to electric flux:

•If B is constant over the surface area A, then
ΦB = BA = BA cosθ
(The scalar or dot product of vectors B & A)
•The SI Unit of Magnetic flux = Weber (Wb):
1 Wb = 1 T·m2.
This figure shows the variables in the flux equation:
ΦB = BA = BA cosθ
•Magnetic Flux is analogous to electric flux:
It
is proportional to the total number of
magnetic field lines passing through the loop.
Faraday’s Law of Induction
• The magnitude of the emf induced in a conducting loop is equal to the rate
at which the magnetic flux through that loop changes with time ,
d B
  , where    B.dA
dt

• If a coil consists of N loops with the same area,


the total induced emf in the
d coil is given by
  N B
dt

d
• In uniform magnetic
 field,
 ( BAthecos
induced
) emf can be
dt
expressed as 13
Conclusions:
To induce an emf we can change, d
   ( BA cos  )
dt
1. The magnitude of B.
2. The area enclosed by the loop
3. The angle between B and the normal to the area
4. Any combination of the above over time.

14
Faraday’s Law of Induction

Faraday’s Law: The average emf in a circuit (with N


loops) equals the rate of change of magnetic flux through
the circuit:

  f  i
 N  N
t t f  ti
Example: EMF induced in a loop
A wire loop of radius 0.30 m lies so that an external magnetic field of magnitude
0.30 T is perpendicular to the loop. The field reverses its direction, and its magnitude
changes to 0.20 T in 1.5 s. Find the magnitude of the average induced emf in the loop
during this time.

B


B 16
The loop is always perpendicular to the field, so the normal to the loop
is parallel to the field, so cos q = 1. The flux is then

  BA  B r 2

Initially the flux is

 i   0.30T    0.30m  =0.085 T  m 2


2

and after the field changes the flux is

 f   0.20T    0.30m  =-0.057 T  m 2


2

The magnitude of the average induced emf is:

0.095 V
17
Example: One way to Induce an emf in a coil
A coil consists of 200 turns of wire having a total resistance of 2.0 . Each turn is a
square of side 18 cm, and a uniform magnetic field directed perpendicular to the
plane of the coil is turned on. If the field changes linearly from 0 to 0.50 T in 0.80s,
what is the magnitude of the induced emf in the coil while the field is changing.
 The area of one turn of the coil is (0.18m)2 = 0.0324 m2.
 The magnetic flux through the coil at t=0 is zero because B=0 at that time.
 At t=0.80s, the magnetic flux through one turn is :

B = BA = (0.50T)(0.0324m2) = 0.0162 T.m2


 Therefore, the magnitude of the induced emf is :

N B 200(0.0162T.m 2  0T.m 2 )


   4.1T.m 2 / s  4.1V
t 0.80s
18
Lenz’s Law
Heinrich Friedrich Emil Lenz

1804 – 1865

Russian physicist
Lenz’s Law
•The minus sign gives the
direction of the induced emf.
 Lenz’s Law:
An induced emf is always
in a direction that OPPOSES
the original change in flux
that caused it.
It is as if the induced emf is producing a virtual bar magnet to oppose
the motion of the bar magnet that is causing the initial changes.
Motion of the magnet increases the flux through the loop pointing to
the right.

The induced current creates a B-field pointing to the left, i.e. a North
pole, to decrease the increased flux.
It is as if the original bar magnet sees an “image” of itself opposing it.
The B-field increases to the right
Example: Lenz’s law
The direction of any magnetic induction effect is such as to oppose the cause
of the effect
For instance: a current produced by an induced emf moves in a direction so
that its magnetic field opposes the original change in flux

S S N
      
N

v N B v B v B v
N S S B
I I I I

Example: If a North pole moves toward the loop in the plane of the page,
in what direction is the induced current?

Since the magnet is moving parallel to the loop,


there is no magnetic flux through the loop.
Thus the induced current is zero.

28
Example 1: A wire loop is being pulled away from a current-carrying wire.
What is the direction of the induced current in the loop?
1) Clockwise
2) Counterclockwise
3) No induced current

On the right side of the wire the magnetic flux is into the page
and decreasing due to the fact that the loop is being pulled
away. By Lenz’s Law, the induced B field will oppose this I
decrease. Thus, the new B field points into the page, which
requires an induced clockwise current to produce such a B field.

Example 2: What is the induced current if the wire loop moves down?

1) Clockwise
2) Counterclockwise
3) No induced current

The magnetic flux through the loop is not changing as it moves


parallel to the wire. Therefore, there is no induced current.
I
29
Example1: A wire loop is being pulled through a uniform magnetic field.
What is the direction of the induced current?
1) Clockwise; 2) Counterclockwise; 3) No induced current x x x x x x x x x
Since the magnetic field is uniform, the magnetic flux through x x x x x x x x x
the loop is not changing. Thus no current is induced. x x x x x x x x x
x x x x x x x x x
Example2: What is the direction of the induced current if the
x x x x x x x x x
B field suddenly increases while the loop is in the region?
1) Clockwise 2) Counterclockwise 3) No induced current

The increasing B field into the page must be countered by an induced flux out of the
page. This can be accomplished by induced current in the counterclockwise direction in
the wire loop.

Example 3: A wire loop is being pulled through x x x x x


a uniform magnetic field that suddenly ends.
What is the direction of the induced current? x x x x x
1) Clockwise 2) Counterclockwise 3) No induced current x x x x x
The B field into the page is disappearing in the loop, so x x x x x
it must be compensated by an induced flux also into the x x x x x
page. This can be accomplished by an induced current
in the clockwise direction in the wire loop.
30
Example : An Exponentially Decaying B
Field
A loop of wire enclosing an area A is
placed in a region where the magnetic field
is perpendicular to the plane of the loop.
The magnitude of B varies in time
according to the expression B = Bmax e-at,
where a is some constant. That is, at t=0
the field is Bmax, and for t>0, the field
decreases exponentially. Find the induced
emf in the loop as a function of time.

31
Solution
 Because B is perpendicular to the plane of the loop, the
magnetic flux thought the loop at time t>0 is : B = BAcos0 =
ABmaxe-at
 Because ABmax and a are constants, the induced emf is :
d B d
   ABmax e at  aABmax e at
dt dt

 This expression indicates that the induced emf decays


exponentially in time.
 Note that the maximum emf occurs at t = 0, where max =
aABmax.

32
Example : A 12.0-cm-diameter wire coil is initially oriented perpendicular to a 1.5
T magnetic field. The loop is rotated so that its plane is parallel to the field
direction in 0.20 s. What is the average induced emf in the loop?

N 1  B  BA cos 
 f   0  
 B   Bf    Bin   BA cos 0  cos 90  BA
 in   90  B BA
  
2r  12.0cm t t
A  r 2
B  1.5T

1.5T    0.12m / 2 
2
 8.5 10  2 V
t  0.20 s 0.20 s
 ?

33
Faraday’s Law: the Generator
A coil rotates with a constant angular speed in a
magnetic field.

EMF   N
t
  AB cos 
but f changes
with time:
  t
Faraday’s Law: the Generator
So the flux also changes with time:
  AB cos   AB cost 
Get the time rate of change (a calculus problem):


  AB sin t 
t
Substitute into Faraday’s Law:


EMF   N  NAB sin t 
t
Faraday’s Law: the Generator
The maximum voltage occurs when n: 
t 
2
EMFmax  NAB

What makes the voltage larger?


more turns in the coil
a larger coil area
a stronger magnetic field
a larger angular speed
Applications of Faraday’s law
Rotating loop:   t
  2f  2 T I
 B  BA cos   BA cos t B

 B BA cos t 
   I
t t

  BA sin t  max


Example:
N  200  max  NBA
B  0.03T
A  100cm 2
  100s 1
 max  ?  max  200  0.03T   100  10 4 m 2  100s 1   6V
37
Example: A generator rotates at 60 Hz in a magnetic field of 0.03
T. It has 1000 turns and produces voltage that is 120 V at a pick.
What is the area of each turn of the coil?

 max  NBA  max


f  60 Hz A
  2f 2fNB
B  0.03T
N  1000
 max  120V
A? 120V
A  10  2 m 2
2  60 Hz 1000  0.03T 

38
EMF resulting from the motion of a conductor
in a constant magnetic field.

39
Motional EMF
As the wire moves,

FB  qv  B

Which sets the positive charges in motion in


the direction of FB and leaves negative
charges behind.

As they accumulate on the bottom, an electric field is set up inside.

In equilibrium,

FB  FE or qvB  qE or E  vB

V  El  Blv 40
Motional EMF in a Circuit
 B  BA  Blx

d B d dx
E    Blx    Bl
dt dt dt

E Blv
E   Blv I 
R R

If the bar is moved with constant velocity,

Fapp  FB  IlB

B 2l 2 v 2 E 2
P  Fapp v  IlB v  
R R
41
example
A rectangular wire loop is pulled thru a uniform B field
penetrating its top half, as shown. The induced current :
1. Current CW Bout
2. Current CW,
3. Current CCW,
4. Current CCW, v
5. No current,
. No current
The motion does not change the magnetic flux, so Faraday’s Law says
there is no induced EMF, or current, or force, or torque.
Of course, if we were pulling at all up or down there would be a force
to oppose that motion.
example
A circuit in the form of a rectangular piece of
wire is pulled away from a long wire carrying
current I in the direction shown in the sketch.
The induced current in the rectangular circuit is
1. Clockwise
2. Counterclockwise
3. Neither, the current is zero
1. Induced current is clockwise

•B due to I is into page; the flux through the circuit due to that
field decreases as the circuit moves away. So the induced
current is clockwise (to make a B into the page)
Example
Consider the circuit .the length of the moving rod is 0.2 m, its speed
is 0.1m/s , the magnetic field-strength is 1T), and the resistance of
the circuit is 0.02Ω)

1.What is the emf generated around the circuit?

2.What current flows around the circuit?


3.What is the magnitude and direction of the force acting on the
moving rod due to the fact that a current is flowing along it?

F  ILB  1 1 0.2  0.2 N

4.What is the power delivered by the applied force?


P  Fv  0.2  0.1  0.02W
P  I  I 2 R
Example : Magnetic Force Acting on a Sliding Bar
The conducting bar illustrated in Figure, of mass m and  length ,
moves on two frictionless parallel rails in the presence of a uniform
magnetic field directed into the page. The bar is given an initial
velocity vi to the right and is released at t=0. Find the velocity of
the bar as a function of time.

46
Figure shows a right view of a bar that can slide ]4[
without friction. The resistor is 6.00 Ω and a 2.50T
magnetic field is directed perpendicularly downward,
.into the paper. Let L= 1.20 m
Calculate the applied force required to move the bar )a(
.to the right at a constant speed of 2.00 m/s
?At what rate is energy delivered to the resistor )b(

47
48
49
Induced nonelectrostatic electric field
d B
dI
 
dt
0 dt
Origin of emf? No motion, moreover no B outside
solenoid, i.e. in the region of a wire loop. Then it
should be E which results in induced current.

 
Fel  dl d B
 q     dt
dI    
0 Fel  qE ,  Fel  dl  0 Nonconcervative force
dt
  d B
 E  dl   dt
 
 E  d l  0 Nonelectrostatic field
B(t) should induce E by independently on the presence of the loop of the wire!
Let’s find E(r).
  d B
 E  dl   dt
1.r  R
dB
R E 2r  r 2

dt
B  ni,   K m  0
n di
E r
2 dt
E
2) r  R
dB
E 2r  R 2

dt
r
n di R 2
R E
2 dt r
Example : Electric Field Induced by a Changing Magnetic
Field in a Solenoid
A long solenoid of radius R has n turns of wire per unit length
and carries a time-varying current that varies sinusoidally as:
I = Imax cos t, where Imax is the maximum current and  is the
angular frequency of the alternating current source Figure. (a)
Determine the magnitude of the induced electric field outside the
solenoid, a distance r > R from its long central axis. (b) What is
the magnitude of the induced elelctric field inside the solenoid, a
distance r from its axis?

52
Solution for (a)
Consider an external point and take the path for our line integral to be a circle
of radius r centered on the solenoid as illustrated in Figure (31.18).
The magnitude of E is constant on this path and that E is tangent to it.
The magnetic flux through the area enclosed by this path is BA = B R2; hence :

  d 2 dB
      
2
E ds ( B R ) R
dt dt
  2 dB (1)
 E  ds  E (2r )  R dt
The magnetic field inside a long solenoid is given by , B = onI.
Substitute I = Imax cos t into this equation and then substitute the result into
Eq. (1), we find that :

53
d
E (2r )  R 2 o nI max (cos t )
dt
 R 2 o nI max  sin t

 o nI max R 2
E sin t (for r > R) (2)
2r

54
Solution for (b)
 For an interior point (r < R), the flux threading an integration loop is given
by Br2.
 Using the same procedure as in part (a), we find that :

dB
E (2r )  r 2  r 2 o nI max  sin t
dt

 o nI max  (3) (for r < R)


E r sin t
2

55
:EXAMPLE

A rectangular coil of 150 loops forms a closed circuit


with a resistance of 5 and measures 0.2 m wide by 0.1
m deep, as shown below. The circuit is placed between
the poles of an electromagnet which is producing a
uniform magnetic field of 40 T. The magnet is switched
off, causing the magnetic field to drop to zero in 2 s.
(The loops are parallel with the faces of the
electromagnet.)

.a) Compute the average induced potential in the circuit


.b) Determine the average current in the circuit
c) Indicate on the drawing the direction in which the induced
.current flows

56
a ]   (0.1)(0.2)[0  40]  0.8T .m 2

d  0.8
  N  (150)[ ]  60V
dt 2
V 60
b] I    12 A
R 5

C] The magnetic field from the magnet is up, but the flux
is decreasing. So the magnetic field from the current
induced in the loop(s) will also be up. Thus the current
.flow is left-to-right across the front of the loop

57

You might also like